Understanding formulas for $E(X∣X < c)$












0












$begingroup$


I have $E(X∣X < c)$ where $X$ is a continuous random variable and $c$ is a given positive real value.



According to this question this is equivalent to:




  1. $E[X|X<c]=int_{-inf}^{c}1-frac{F(x)}{F(c)}dx$


On this other question we have both:




  1. $mathrm E(Xmid X < c)=frac{mathrm E(Xcdotmathbf 1_{X < c})}{mathrm P(Xgt c)}$

  2. $E(X|X<c) = frac{int_x x f(x) I(x<c) dx}{int_x f(x) I(x<c) dx}$


Yet, starting from the definition of conditional expectation, I couldn't get to show any of the three equations.
I am particularly interested in the first one.



Can you help me understand how to get it? (And or the others)



Thanks.










share|cite|improve this question









$endgroup$












  • $begingroup$
    The first one looks almost right when you integrate by parts. The third one is a roundabout way of integrating from$-infty$ to $c$ for both the the numerator and the denominator. The second seems to have an error for the denominator.
    $endgroup$
    – herb steinberg
    Dec 2 '18 at 23:27










  • $begingroup$
    Ps. Ensure that you use the definition for expectation conditioned over an event, rather than a random variable.
    $endgroup$
    – Graham Kemp
    Dec 3 '18 at 0:05
















0












$begingroup$


I have $E(X∣X < c)$ where $X$ is a continuous random variable and $c$ is a given positive real value.



According to this question this is equivalent to:




  1. $E[X|X<c]=int_{-inf}^{c}1-frac{F(x)}{F(c)}dx$


On this other question we have both:




  1. $mathrm E(Xmid X < c)=frac{mathrm E(Xcdotmathbf 1_{X < c})}{mathrm P(Xgt c)}$

  2. $E(X|X<c) = frac{int_x x f(x) I(x<c) dx}{int_x f(x) I(x<c) dx}$


Yet, starting from the definition of conditional expectation, I couldn't get to show any of the three equations.
I am particularly interested in the first one.



Can you help me understand how to get it? (And or the others)



Thanks.










share|cite|improve this question









$endgroup$












  • $begingroup$
    The first one looks almost right when you integrate by parts. The third one is a roundabout way of integrating from$-infty$ to $c$ for both the the numerator and the denominator. The second seems to have an error for the denominator.
    $endgroup$
    – herb steinberg
    Dec 2 '18 at 23:27










  • $begingroup$
    Ps. Ensure that you use the definition for expectation conditioned over an event, rather than a random variable.
    $endgroup$
    – Graham Kemp
    Dec 3 '18 at 0:05














0












0








0





$begingroup$


I have $E(X∣X < c)$ where $X$ is a continuous random variable and $c$ is a given positive real value.



According to this question this is equivalent to:




  1. $E[X|X<c]=int_{-inf}^{c}1-frac{F(x)}{F(c)}dx$


On this other question we have both:




  1. $mathrm E(Xmid X < c)=frac{mathrm E(Xcdotmathbf 1_{X < c})}{mathrm P(Xgt c)}$

  2. $E(X|X<c) = frac{int_x x f(x) I(x<c) dx}{int_x f(x) I(x<c) dx}$


Yet, starting from the definition of conditional expectation, I couldn't get to show any of the three equations.
I am particularly interested in the first one.



Can you help me understand how to get it? (And or the others)



Thanks.










share|cite|improve this question









$endgroup$




I have $E(X∣X < c)$ where $X$ is a continuous random variable and $c$ is a given positive real value.



According to this question this is equivalent to:




  1. $E[X|X<c]=int_{-inf}^{c}1-frac{F(x)}{F(c)}dx$


On this other question we have both:




  1. $mathrm E(Xmid X < c)=frac{mathrm E(Xcdotmathbf 1_{X < c})}{mathrm P(Xgt c)}$

  2. $E(X|X<c) = frac{int_x x f(x) I(x<c) dx}{int_x f(x) I(x<c) dx}$


Yet, starting from the definition of conditional expectation, I couldn't get to show any of the three equations.
I am particularly interested in the first one.



Can you help me understand how to get it? (And or the others)



Thanks.







probability conditional-expectation






share|cite|improve this question













share|cite|improve this question











share|cite|improve this question




share|cite|improve this question










asked Dec 2 '18 at 21:51









jpuertajpuerta

11




11












  • $begingroup$
    The first one looks almost right when you integrate by parts. The third one is a roundabout way of integrating from$-infty$ to $c$ for both the the numerator and the denominator. The second seems to have an error for the denominator.
    $endgroup$
    – herb steinberg
    Dec 2 '18 at 23:27










  • $begingroup$
    Ps. Ensure that you use the definition for expectation conditioned over an event, rather than a random variable.
    $endgroup$
    – Graham Kemp
    Dec 3 '18 at 0:05


















  • $begingroup$
    The first one looks almost right when you integrate by parts. The third one is a roundabout way of integrating from$-infty$ to $c$ for both the the numerator and the denominator. The second seems to have an error for the denominator.
    $endgroup$
    – herb steinberg
    Dec 2 '18 at 23:27










  • $begingroup$
    Ps. Ensure that you use the definition for expectation conditioned over an event, rather than a random variable.
    $endgroup$
    – Graham Kemp
    Dec 3 '18 at 0:05
















$begingroup$
The first one looks almost right when you integrate by parts. The third one is a roundabout way of integrating from$-infty$ to $c$ for both the the numerator and the denominator. The second seems to have an error for the denominator.
$endgroup$
– herb steinberg
Dec 2 '18 at 23:27




$begingroup$
The first one looks almost right when you integrate by parts. The third one is a roundabout way of integrating from$-infty$ to $c$ for both the the numerator and the denominator. The second seems to have an error for the denominator.
$endgroup$
– herb steinberg
Dec 2 '18 at 23:27












$begingroup$
Ps. Ensure that you use the definition for expectation conditioned over an event, rather than a random variable.
$endgroup$
– Graham Kemp
Dec 3 '18 at 0:05




$begingroup$
Ps. Ensure that you use the definition for expectation conditioned over an event, rather than a random variable.
$endgroup$
– Graham Kemp
Dec 3 '18 at 0:05










0






active

oldest

votes











Your Answer





StackExchange.ifUsing("editor", function () {
return StackExchange.using("mathjaxEditing", function () {
StackExchange.MarkdownEditor.creationCallbacks.add(function (editor, postfix) {
StackExchange.mathjaxEditing.prepareWmdForMathJax(editor, postfix, [["$", "$"], ["\\(","\\)"]]);
});
});
}, "mathjax-editing");

StackExchange.ready(function() {
var channelOptions = {
tags: "".split(" "),
id: "69"
};
initTagRenderer("".split(" "), "".split(" "), channelOptions);

StackExchange.using("externalEditor", function() {
// Have to fire editor after snippets, if snippets enabled
if (StackExchange.settings.snippets.snippetsEnabled) {
StackExchange.using("snippets", function() {
createEditor();
});
}
else {
createEditor();
}
});

function createEditor() {
StackExchange.prepareEditor({
heartbeatType: 'answer',
autoActivateHeartbeat: false,
convertImagesToLinks: true,
noModals: true,
showLowRepImageUploadWarning: true,
reputationToPostImages: 10,
bindNavPrevention: true,
postfix: "",
imageUploader: {
brandingHtml: "Powered by u003ca class="icon-imgur-white" href="https://imgur.com/"u003eu003c/au003e",
contentPolicyHtml: "User contributions licensed under u003ca href="https://creativecommons.org/licenses/by-sa/3.0/"u003ecc by-sa 3.0 with attribution requiredu003c/au003e u003ca href="https://stackoverflow.com/legal/content-policy"u003e(content policy)u003c/au003e",
allowUrls: true
},
noCode: true, onDemand: true,
discardSelector: ".discard-answer"
,immediatelyShowMarkdownHelp:true
});


}
});














draft saved

draft discarded


















StackExchange.ready(
function () {
StackExchange.openid.initPostLogin('.new-post-login', 'https%3a%2f%2fmath.stackexchange.com%2fquestions%2f3023262%2funderstanding-formulas-for-ex%25e2%2588%25a3x-c%23new-answer', 'question_page');
}
);

Post as a guest















Required, but never shown

























0






active

oldest

votes








0






active

oldest

votes









active

oldest

votes






active

oldest

votes
















draft saved

draft discarded




















































Thanks for contributing an answer to Mathematics Stack Exchange!


  • Please be sure to answer the question. Provide details and share your research!

But avoid



  • Asking for help, clarification, or responding to other answers.

  • Making statements based on opinion; back them up with references or personal experience.


Use MathJax to format equations. MathJax reference.


To learn more, see our tips on writing great answers.




draft saved


draft discarded














StackExchange.ready(
function () {
StackExchange.openid.initPostLogin('.new-post-login', 'https%3a%2f%2fmath.stackexchange.com%2fquestions%2f3023262%2funderstanding-formulas-for-ex%25e2%2588%25a3x-c%23new-answer', 'question_page');
}
);

Post as a guest















Required, but never shown





















































Required, but never shown














Required, but never shown












Required, but never shown







Required, but never shown

































Required, but never shown














Required, but never shown












Required, but never shown







Required, but never shown







Popular posts from this blog

Plaza Victoria

In PowerPoint, is there a keyboard shortcut for bulleted / numbered list?

How to put 3 figures in Latex with 2 figures side by side and 1 below these side by side images but in...